Quantcast
  • Register
PhysicsOverflow is a next-generation academic platform for physicists and astronomers, including a community peer review system and a postgraduate-level discussion forum analogous to MathOverflow.

Welcome to PhysicsOverflow! PhysicsOverflow is an open platform for community peer review and graduate-level Physics discussion.

Please help promote PhysicsOverflow ads elsewhere if you like it.

News

PO is now at the Physics Department of Bielefeld University!

New printer friendly PO pages!

Migration to Bielefeld University was successful!

Please vote for this year's PhysicsOverflow ads!

Please do help out in categorising submissions. Submit a paper to PhysicsOverflow!

... see more

Tools for paper authors

Submit paper
Claim Paper Authorship

Tools for SE users

Search User
Reclaim SE Account
Request Account Merger
Nativise imported posts
Claim post (deleted users)
Import SE post

Users whose questions have been imported from Physics Stack Exchange, Theoretical Physics Stack Exchange, or any other Stack Exchange site are kindly requested to reclaim their account and not to register as a new user.

Public \(\beta\) tools

Report a bug with a feature
Request a new functionality
404 page design
Send feedback

Attributions

(propose a free ad)

Site Statistics

205 submissions , 163 unreviewed
5,047 questions , 2,200 unanswered
5,345 answers , 22,709 comments
1,470 users with positive rep
816 active unimported users
More ...

  Continuous Transition of Degrees of Freedom in Thermodynamics

+ 1 like - 0 dislike
664 views

(Original on SE https://physics.stackexchange.com/questions/459086/continuous-transition-of-degrees-of-freedom-in-thermodynamics-with-simple-exampl, though I usually get better answers here.)

In thermodynamics books I have read, I have often come across statements about how certain degrees of freedom are relevant only at certain temperatures (such as the vibration degrees of freedom of some molecules only being relevant in certain ranges), but I can't recall a convincing quantitative analysis of this. I tried to set up a simple example to explore this issue, but I'm unsure what goes wrong (though I haven't thought TOO much about it).  

We know that for a free particle in one dimension at finite temperature, the partition function is given by:

$$Z(\beta)=\frac{L}{h}\int \text{d}p e^{-\beta p^2/2m}=\frac{L}{h}\sqrt{\frac{2 \pi m}{\beta}}$$

And then our expected energy is just:

$$\langle E \rangle =\frac{1}{2}k_B T$$

Which we'd expect for a particle with one degree of freedom.

On the other hand, the partition function for a particle in a 1-D harmonic potential is:

$$Z(\beta)=\frac{1}{h}\int \text{d}x \int \text{d}p e^{-\beta(kx^2+p^2/m)/2}=\frac{2\pi}{\beta h\omega}$$

Which gives the expected energy:

$$\langle E \rangle = k_BT$$

Here's my problem. If we take a limit of the spring constant to zero ($k \rightarrow 0$), doesn't that just correspond to a free particle? The average energies depend ONLY on temperature, so where exactly does this limit come in?

Even though this is a relatively simple example, I encounter similar problems when I try to work out when and how degree of freedom fails to be "relevant" in the examples alluded to in textbooks.

asked Feb 5, 2019 in Theoretical Physics by connornm777 (30 points) [ no revision ]

1 Answer

+ 1 like - 0 dislike

The answer on SE is quite consistent, but it speaks of classical partition functions of particles "attached" to the coordinate system origin with a quadratic potential.

If your question is about seeing certain degrees of freedom getting "irrelevant", you must consider quantum mechanics. There the energy levels are discrete rather than continuous, so the occupation number for a given energy level $E_n$ is proportional to $\text{e}^{-E_n/k_B T}$ and the partition function is a sum over them. In particular, for a quantum harmonic oscillator at "low" temperatures the contribution of the first excited state $\text{e}^{-E_1/k_B T}$ with respect to the ground state $\text{e}^{-E_0/k_B T}$ is exponentially small: $\propto(1+\text{e}^{-(E_1-E_0)/k_B T})$ due to inequality $(E_1-E_0)/k_B T\gg 1$. They say, the higher energy levels (or the corresponding degrees of freedom) are "frozen" in this limit. If the "oscillator" is a vibrational motion of a two-atomic molecule, then its ground state corresponds to a free point-like particle, for which $\langle E\rangle=0.5k_B T$, as if there were no vibrational degree of freedom at all.

As the enrgy levels $E_n$ contain all the necessary constants, including $\hbar$ and $k$, you can analyse where a quantum oscillator becomes similar to the classical one.

answered Feb 6, 2019 by Vladimir Kalitvianski (102 points) [ revision history ]
edited Feb 7, 2019 by Vladimir Kalitvianski

Your answer

Please use answers only to (at least partly) answer questions. To comment, discuss, or ask for clarification, leave a comment instead.
To mask links under text, please type your text, highlight it, and click the "link" button. You can then enter your link URL.
Please consult the FAQ for as to how to format your post.
This is the answer box; if you want to write a comment instead, please use the 'add comment' button.
Live preview (may slow down editor)   Preview
Your name to display (optional):
Privacy: Your email address will only be used for sending these notifications.
Anti-spam verification:
If you are a human please identify the position of the character covered by the symbol $\varnothing$ in the following word:
p$\hbar$ysicsOv$\varnothing$rflow
Then drag the red bullet below over the corresponding character of our banner. When you drop it there, the bullet changes to green (on slow internet connections after a few seconds).
Please complete the anti-spam verification




user contributions licensed under cc by-sa 3.0 with attribution required

Your rights
...